• Anúncio Global
    Respostas
    Exibições
    Última mensagem

QUESTÃO

QUESTÃO

Mensagempor GABRIELA » Ter Set 08, 2009 16:32

SE
\begin{pmatrix}
   A & B & C \\ 
   M & N & P \\
   Z & T & U
\end{pmatrix}\: = R (R\neq0)
Então por que
\begin{pmatrix}
   2A & 2B & 2C \\ 
   M & N & P \\
   3Z & 2T & 3U
\end{pmatrix}\
=6R
GABRIELA
Usuário Parceiro
Usuário Parceiro
 
Mensagens: 73
Registrado em: Seg Ago 31, 2009 17:31
Formação Escolar: ENSINO MÉDIO
Andamento: cursando

Re: QUESTÃO

Mensagempor Molina » Ter Set 08, 2009 19:08

Boa tarde, Gabriela.

Confirma: O elemento {a}_{32} da segunda matriz não seria 3T?

Isso é uma propriedade de determinante.
Se multiplicarmos uma linha por algum escalar o determinante será multiplicado pelo mesmo escalar. Como neste exemplo multiplicamos por 2 na primeira linha e por 3 na segunda, 2*3=6 e foi esta variação que o determinante R sofreu.

Até mais, :y:
Diego Molina | CV | FB | .COM
Equipe AjudaMatemática.com


"Existem 10 tipos de pessoas: as que conhecem o sistema binário e as que não conhecem."
Avatar do usuário
Molina
Colaborador Moderador - Professor
Colaborador Moderador - Professor
 
Mensagens: 1551
Registrado em: Dom Jun 01, 2008 14:10
Formação Escolar: GRADUAÇÃO
Área/Curso: Licenciatura em Matemática - UFSC
Andamento: formado

Re: QUESTÃO

Mensagempor GABRIELA » Ter Set 08, 2009 21:21

molina escreveu:Boa tarde, Gabriela.

Confirma: O elemento {a}_{32} da segunda matriz não seria 3T?

Isso é uma propriedade de determinante.
Se multiplicarmos uma linha por algum escalar o determinante será multiplicado pelo mesmo escalar. Como neste exemplo multiplicamos por 2 na primeira linha e por 3 na segunda, 2*3=6 e foi esta variação que o determinante R sofreu.

Até mais, :y:

GABRIELA escreveu:SE
\begin{pmatrix}
A & B & C \\ 
M & N & P \\
Z & T & U
\end{pmatrix}\: = R (R\neq0)
Então por que
\begin{pmatrix}
2A & 2B & 2C \\ 
M & N & P \\
3Z & 2T & 3U
\end{pmatrix}\
=6R

Verdade! É 3t errei na digitação,me empolguei com o 2.kkk
Mas entendi o pq.Veleu! :y:
GABRIELA
Usuário Parceiro
Usuário Parceiro
 
Mensagens: 73
Registrado em: Seg Ago 31, 2009 17:31
Formação Escolar: ENSINO MÉDIO
Andamento: cursando


Voltar para Matrizes e Determinantes

 



  • Tópicos relacionados
    Respostas
    Exibições
    Última mensagem

Quem está online

Usuários navegando neste fórum: Nenhum usuário registrado e 25 visitantes

 



Assunto: Unesp - 95 Números Complexos
Autor: Alucard014 - Dom Ago 01, 2010 18:22

(UNESP - 95) Seja L o Afixo de um Número complexo a=\sqrt{8}+ i em um sistema de coordenadas cartesianas xOy. Determine o número complexo b , de módulo igual a 1 , cujo afixo M pertence ao quarto quadrante e é tal que o ângulo LÔM é reto.


Assunto: Unesp - 95 Números Complexos
Autor: MarceloFantini - Qui Ago 05, 2010 17:27

Seja \alpha o ângulo entre o eixo horizontal e o afixo a. O triângulo é retângulo com catetos 1 e \sqrt{8}, tal que tg \alpha = \frac{1}{sqrt{8}}. Seja \theta o ângulo complementar. Então tg \theta = \sqrt{8}. Como \alpha + \theta = \frac{\pi}{2}, o ângulo que o afixo b formará com a horizontal será \theta, mas negativo pois tem de ser no quarto quadrante. Se b = x+yi, então \frac{y}{x} = \sqrt {8} \Rightarrow y = x\sqrt{8}. Como módulo é um: |b| = \sqrt { x^2 + y^2 } = 1 \Rightarrow x^2 + y^2 = 1 \Rightarrow x^2 + 8x^2 = 1 \Rightarrow x = \frac{1}{3} \Rightarrow y = \frac{\sqrt{8}}{3}.

Logo, o afixo é b = \frac{1 + i\sqrt{8}}{3}.